LSAT and Law School Admissions Forum

Get expert LSAT preparation and law school admissions advice from PowerScore Test Preparation.

User avatar
 Dave Killoran
PowerScore Staff
  • PowerScore Staff
  • Posts: 5852
  • Joined: Mar 25, 2011
|
#44165
Complete Question Explanation
(The complete setup for this game can be found here: lsat/viewtopic.php?t=9437)

The correct answer choice is (A)

To solve this question, we must first determine which fish species should be included; that is, which fish species are necessary to allow other fish species to be chosen. In this game, one fish species stands out as the strongest candidate: O. Because O is a necessary condition for M, N, and P, if O is not selected, then via the contrapositive M, N, and P cannot be selected, a loss of four fish species (O, M, N, P). On the other hand, if O is selected, then O, P, and the choice of M or N can be selected, a total of three selected fish species (O, P, M/N). Clearly, the selection of O has a dramatic positive effect on the maximum number of fish species selected. Hence, if we accept that O should be one of the selected fish species, then from the first rule we can establish that K cannot be selected. Answer choice (A) is therefore correct.

Get the most out of your LSAT Prep Plus subscription.

Analyze and track your performance with our Testing and Analytics Package.